LSAT PrepTest 30 Section 3 Question 4 Explanation | Logic Games

I didn't write the following blog post. It was already on the blog when I took over the URL. The following blog post may contain mistakes. -Steve

***


This is a Logic Games question from the December 1999 LSAT.


This is a straightforward "loose sequencing" game, which means it can be solved with the same tools we've always used for Logic Games. As always, we'll make a diagram using slots with symbols.

Let's create a setup that satisfies all the rules, which will allow us to eliminate choices by comparing them with our diagram. Many choices here are arbitrary, but we just have to make some choice.

Let's have G go first, and since the fourth rule tells us G requires F and P, let's put them 2nd and 3rd:

G F P

1 2 3 4 5 6

Now, since F left a message, we know P and T did too, because of the fifth rule. We also know that P's messages come before all of T's. P is already in, so let's place T right after P:

G F P T

1 2 3 4 5 6

The last rule tells us that P requires H and L, with H's messages coming before L's. Let's put H and L in as the last two:

G F P T H L

1 2 3 4 5 6

This fits all the rules. Now, let's use this valid diagram to eliminate some choices. Remember, if a choice conflicts with our diagram, it doesn't always have to be true, so we can eliminate it.

A) True in our diagram, so we can keep it.

B) Same as A.

C) Same as A.

D) Correct. Not true in our diagram, so we can eliminate it.

E) Same as A.

Remember:

1) Even if a game seems a little unusual, you can almost always rely on our usual slot diagram. The one exception that comes to mind is an old game involving airplane routes (on Zephyr Airlines). Because these game-types rarely come up on modern PrepTests, I don't cover them on the blog. Making a slot diagram almost always works.

2) Make a diagram that follows the rules, by making arbitrary choices to find a setup that works. Then, check the choices against it when the question asks what "must be true," "could be true," "cannot be true," etc.



11 comments:

  1. Can someone explain the easiest way to conquer this game please?

    Test # 4, Questions 1-6

    John receives one grade for each of the following six courses: economics, geology, history, Italian, physics, and Russian. From highest to lowest, the possible grades are A, B, C, D, and E. E is the only failing grade. Two letter grades are consecutive if and only if they are adjacent in the alphabet.
    John's grades in geology and physics are consecutive.
    His grades in Italian and Russian are consecutive.
    He receives a higher grade in economics than in history.
    He receives a higher grade in geology than in physics.

    1. If John receives the same grade in economics and Italian, and if he fails Russian, which one of the following must be true?
    (A) John's geology grade is a B.
    (B) John's history grade is a D.
    (C) John's history grade is an E
    (D) John's physics grade is a B.
    (E) John's physics grade is a C.

    2. If John passes all his courses and receives a higher grade in geology than in either language, which one of the following must be ture?
    (A) He receives exactly one A.
    (B) He receives exactly one B.
    (C) He receives exactly two Bs.
    (D) He receives at least one B and at least one C.
    (E) He receives at least one C and at least one D.

    3. If John receives a higher grade in physics than in economics and receives a higher grade in economics than in either language, which one of the following allows all six of his grades to be determined?
    (A) His grade in history is D.
    (B) His grade in Italian is D.
    (C) His grades in history and Italian are identical.
    (D) His grades in history and Russian are identical.
    (E) His grade in history is higher than his grade in Russian.

    4. If John receives a higher grade in physics than in economics and receives a higher grade in history than in Italian, exactly how many of his grades can be determined?
    (A) 2
    (B) 3
    (C) 4
    (D) 5
    (E) 6

    5. Assume that John's grade in physics is higher than his grade in Italian and consecutive with it and that his grades in Russian and physics differ. Which one of the following must be ture?
    (A) John receives both an A and a B.
    (B) John receives both an A and a C.
    (C) John receives both a B and a D.
    (D) John receives both a B and an E.
    (E) John receives both a D and an E.

    6. Assume that John receives a lower grade in economics than in physics. He must have failed at least one course if which one of the following is also ture?
    (A) He receives a lower grade in Italian than in economics.
    (B) He receives a lower grade in Italian than in physics.
    (C) He receives a lower grade in physics than in Italian.
    (D) He receives a lower grade in Russian than in economics.
    (E) He receives a lower grade in Russian than in history.








    Answers are
    1)C
    2)D
    3)E
    4)E
    5)C
    6)E

    ReplyDelete
  2. With the Test#4 game, the best way to do it (IMO) is just to dive right in and use the question stems along with the rules to help create the hypotherical situations.

    For example, in 1) we know r = E. This means that i = D. The question stem also tells us that i = e, so e = D. Also we know e > h. Since the only grade less than a D is an E, h = E.

    That's why C is right.

    ReplyDelete
  3. I'd rather see arguments or reading comp,personally.

    ReplyDelete
  4. How do you figure out No. 3 above? Process of Elimination?

    ReplyDelete
  5. for number 3 we know that:
    G->P>E>H and I and R

    since I and R can't be the same grade, weknow that its
    G->P>E>H and I and R
    A B C d/e d/e d/e

    choice E is the only one that resolves H, I and R.

    ReplyDelete
  6. Hi,

    I'm struggling with an argument that I simply cannot decipher for the life of me. Could anyone perhaps explain Test 44, Section 2, Question 22? My test book says the answer is E but I can't figure out why. pasted below:

    thanks.

    All the evidence so far gathered fits both Dr. Grippen's theory and Professor Heissman's. However, the predictions that these theories make about the result of the planned experiment cannot both be true. Therefore, the result of this experiment will confirm one of these theories at the expense of the other.

    The argument above exhibits an erroneous pattern of reasoning most similar to that exhibited by which one of the following?

    (a)Dave and Jane both think they know how to distinguish beech trees from elms, but when they look at trees together they often disagree. Therefor, at least one of them must have an erroneous method.

    (b) Although David thinks the tree they saw was a beech, Jane thinks it was an elm. Jane's description of the tree's features is consistent with her opinion, so this description must be inconsistent with David's view.

    (c) David and Jane have been equally good at identifying trees so far. But David says this one is an elm, whereas Jane is unsure. Therefore, if this tree turns out to be an elm, we'll know David is better.

    (d)David thinks that there are more beeches than elms in this forest. Jane thinks he is wrong. The section of the forest we examined was small, but examination of the whole forest would either confirm David's view or disprove it.

    (e) David thinks this tree is a beech. Jane thinks it is an elm. Maria, unlike David or Jane, is expert at tree identification, so when Maria gives her opinion it will verify either David's or Jane's opinion.

    ReplyDelete
  7. I think the key to answering this question correctly is to realize there is a 3rd party involved (the experiment / Maria, an expert) which verifies one opinion being correct and the other's wrong. To realize that one has to be right and one has to be wrong is not enough!

    ReplyDelete
  8. I think the error in that last question is that both of them could be incorrect. The stem doesn't say one of them absolutely has to be right.

    The result of the experiment could prove both of the docotrs wrong,which would NOT "confirm one at the expense of the other".

    ReplyDelete
  9. Hiya! How about adding some new questions this last week before the Oct. LSAT?

    ReplyDelete
  10. this can't be the correct answer because theodore can't have left a message.

    two reasons...
    1. pasquale left the last message
    2. all of pasquaele's messages must precede those of theodore.

    if pasquale left the last message then theodore cannot have left a message...

    ReplyDelete
  11. Pasquale didn't necessarily leave the last message. It says that if Hildy went first, then Pasquale is last. We're assuming that Hildy did not go first.

    ReplyDelete